Doctor: While a few alternative medicines have dangerous side effects, some, such as many herbs, have been proven sa...

Meredith on September 10, 2019

Choice A

I eliminated A due to the use of the quantifier: many = some = could be just 1 how can we ignore that in this case?

Reply
Create a free account to read and take part in forum discussions.

Already have an account? log in

SamA on September 17, 2019

Hello @Meredith,

Let's break down the problem.

Premise: Patients will not be harmed by consuming safe herbs
Premise: Patients might be helped by herbs

Conclusion: Advocates of herbal remedies should be allowed to prescribe them

Here is why the quantifier "many" doesn't disqualify answer choice A. The doctor's argument relies upon the premise that "since their patients will not be harmed." Answer choice A gives an example of harm that can befall many patients. No matter how many patients this affects, it attacks a key premise from the argument. "Many" actually weakens the doctor's argument more than would "some" or "a few." We don't have to ignore the quantifier, we just do not need every single patient to be harmed in order to weaken the argument.